Group exercise for rotations of regular n-gon objects

Click For Summary
The discussion revolves around the generation sets for regular n-gon objects, specifically for n=4, n=5, and n=6. It is clarified that the sets presented for n=4 and n=6 are incorrect as they do not generate the full cyclic groups C4 and C6, while the set for n=5 is correct. The conversation emphasizes the need to identify all minimal generating sets for these cases, noting that the inclusion of the identity element may be redundant. Participants agree on the necessity to correct the earlier cases and explore additional generating sets for n=4, n=5, and n=6.
LCSphysicist
Messages
644
Reaction score
162
Homework Statement
\
Relevant Equations
.
1608184421413.png

The doubt is about B and C.

b)
n = 4, $C = {I,e^{2\pi/4}}
n = 5, $C = {I,e^{2\pi/5}}
n = 6, $C = {I,e^{2\pi/6}}

Is this right?

c)
I am not sure what does he wants...
 
Last edited by a moderator:
Physics news on Phys.org
Well, let's start by case ##n=4## in part b):
What you have is essentially
$$\langle e, r^2\rangle$$
But, from part a), if you did it correctly, you should know that ##r^4=e## and therefore
$$\langle e, r^2\rangle = \{e, r^2\} \neq C_4$$
So no, what you have is not a generating set.
The same is true for case ##n=6## where
$$\langle e, r^2\rangle = \{e, r^2, r^4\}\neq C_6$$
You did it correctly for ##n=5## where
$$\langle e, r^2\rangle = \{e, r^2, r^4, r, r^3\} = G_5$$

Now, you should first correct the cases ##n=4,6## and then, for case ##n=5## is this the only minimal generating set?
 
  • Like
Likes Delta2
Gaussian97 said:
Well, let's start by case ##n=4## in part b):
What you have is essentially
$$\langle e, r^2\rangle$$
But, from part a), if you did it correctly, you should know that ##r^4=e## and therefore
$$\langle e, r^2\rangle = \{e, r^2\} \neq C_4$$
So no, what you have is not a generating set.
The same is true for case ##n=6## where
$$\langle e, r^2\rangle = \{e, r^2, r^4\}\neq C_6$$
You did it correctly for ##n=5## where
$$\langle e, r^2\rangle = \{e, r^2, r^4, r, r^3\} = G_5$$

Now, you should first correct the cases ##n=4,6## and then, for case ##n=5## is this the only minimal generating set?
Hello. I think i do not understand yet the b. You said that what i wrote in the first and third case is essentially the same as $\langle e, r^2\rangle$, but my aim was to be the same as $\langle e, r\rangle$. Can't i call it equal r? So technically $r^4 = e$
 
Herculi said:
Hello. I think i do not understand yet the b. You said that what i wrote in the first and third case is essentially the same as $\langle e, r^2\rangle$, but my aim was to be the same as $\langle e, r\rangle$. Can't i call it equal r? So technically $r^4 = e$
Oh yes, sorry my mistake, I read the exponential of ##4\pi## instead of the ##2\pi##. Then yes what you wrote is just ##\langle e, r \rangle## and it works for ##C_4## and ##C_6##, but as I have shown you for ##C_5##, ##\langle e, r^2\rangle## is also a minimal generating set. So now the question is, are there more minimal generating sets? You need to find them all.
 
I don't see why you have I in the generating sets. Isn't it redundant? Indeed, it mentions sets of size 2 for n=6, and I can see what they are, but your system would make them size 3.
Your list is incomplete for each of 4, 5, 6.
 
Question: A clock's minute hand has length 4 and its hour hand has length 3. What is the distance between the tips at the moment when it is increasing most rapidly?(Putnam Exam Question) Answer: Making assumption that both the hands moves at constant angular velocities, the answer is ## \sqrt{7} .## But don't you think this assumption is somewhat doubtful and wrong?

Similar threads

  • · Replies 16 ·
Replies
16
Views
2K
  • · Replies 1 ·
Replies
1
Views
2K
  • · Replies 1 ·
Replies
1
Views
2K
  • · Replies 3 ·
Replies
3
Views
2K
Replies
3
Views
2K
  • · Replies 1 ·
Replies
1
Views
2K
  • · Replies 7 ·
Replies
7
Views
2K
  • · Replies 12 ·
Replies
12
Views
2K
  • · Replies 1 ·
Replies
1
Views
1K
Replies
4
Views
2K